in a class ,ratio of the number of girls to boys is13:7 , find the percentage of boy​

Answers

Answer 1
I’m pretty sure it’s 35% of boys in the class cuz 13+7=20 which is the total number of students in the class and 20x.35=7

Related Questions

Which table of values could be generated by the equation 10x+5y=15? (Will give brainlest and 21 points)

Answers

Answer:

(For the image) A

Step-by-step explanation:

 The angle of elevation to a nearby tree from a point on the ground is measured to be 65°. How tall is the tree if the point on the ground is 92 feet from the tree? Round your answer to the nearest hundredth of a
foot if necessary.

Answers

Answer:

197.3 feet

Step-by-step explanation:

197.295 rounded to the nearest hundredth is 197.30 or 197.3

The height of the tree if the point on the ground from the tree is 92 feet will be 197.29 feet.

What is a right-angle triangle?

It is a type of triangle in which one angle is 90 degrees and it follows the Pythagoras theorem and we can use the trigonometry function. The Pythagoras is the sum of the square of two sides is equal to the square of the longest side.

The angle of elevation to a nearby tree from a point on the ground is measured to be 65°.

The height of the tree if the point on the ground from the tree is 92 feet. Then we have

Let h be the height of the tree. Then we have

[tex]\tan 65^o = \dfrac{h}{92}\\\\ 2.1445\ = \dfrac{h}{92}[/tex]

Then we have

[tex]\rm h = 2.1445 \times 92\\\\h = 197.29\ ft[/tex]

More about the right-angle triangle link is given below.

https://brainly.com/question/3770177

Solve the equation -9x -21= 20x -87

Answers

Answer:

66/29

Step-by-step explanation:

-9x - 21 = 20x - 87

Add 87 to both sides to cancel out the -87 on the right side:

-9x + 66 = 20x

Add 9x to both sides to cancel out the -9x on the left side:

66 = 29x

Divide both sides by 29 to isolate the variable:

66/29 or 2.275862....

A car travels 600 km in 6 hours. at what rate of speed is the car traveling?

Answers

Step-by-step explanation:

100Km/hours......V/T

Write each power as a single power, then evaluate (9

Answers

Answer:

Step-by-step explanation:

a.  〖(9^8)〗^0 = 1

b.  〖〖[(-2)〗^4]〗^2 = 256

c.  〖–(3^2)〗^3 = -729

Find the length of side BC give your answer to three significant figures

Answers

Answer:

19.4 cm

Step-by-step explanation:

Hi there!

This is a right triangle. We're given an angle, the side adjacent to the angle and we're solving for the hypotenuse. Given this information, we can use the cosine ratio:

[tex]cos\theta=\frac{adj}{hyp}[/tex]

Plug in the given angle and side

[tex]cos71=\frac{6.3}{BC}\\BC=\frac{6.3}{cos71} \\BC=19.4[/tex]

Therefore, the length of BC is 19.4 cm when rounded to 3 significant figures.

I hope this helps!

Please HELP!! Will mark brainliest!!!

Answers

Answer:

x=6

y=5

Step-by-step explanation:

15x and 90 are vertical angles and vertical angles are equal

15x = 90

Divide by 15

15x/15 = 90/15

x = 6

15x and 5x+12y are corresponding angles and corresponding angles are equal when the lines are parallel

15x= 5x+12y

Subtract 5x

15x-5x = 5x+12y-5x

10x = 12y

10(6) = 12y

60 = 12y

Divide by 12

60/12 = 12y/12

5 = y

Isabella runs 3 times per week. She ran for 15 minutes on Monday and 17 minutes on Wednesday. Her coach told her that she had run 80 percent of her goal that week. How many more minutes does she need to run to meet her goal for the week?

Answers

Answer

8 More minutes

Step-by-step explanation:

15+ 17 = 32

32/.8 = 40

40 - 32 = 8

A triangle has an area of 18ft squared. List all the possible positive integers that could represent its base and height. Use sentences to describe your process.

Answers

Answer:

1ft x 36ft

2ft x 18ft

3ft x 12ft

4ft x 9ft

6ft x 6ft

Step-by-step explanation:

A triangle is a three-sided polygon with three edges and three vertices. the sum of angles in a triangle is 180 degrees

Area of a triangle = 1/2 x base x height

If area is 18ft², then the dimensions of base x height = 18 x 2 = 36

all the possible positive integers that could represent its base and height can be determined be known by determining the factors of 36

factors of 36 = 1, 2, 3, 4, 6, 9, 12, 18, and 36

Possible dimensions of base and heights are :

1ft x 36ft

2ft x 18ft

3ft x 12ft

4ft x 9ft

6ft x 6ft

The population of China is about 1.4 x 10^9 people. Julie grew up in a town that had a population of about 7,000 people. How many times greater is the population of China than Julie's hometown?


Please write your answer in standard notation

Answers

Answer:

200,000 times

Step-by-step explanation:

The population of China is about 1.4 × 10⁹ . And the hometown of Julie has a population of 7,000 people . We need to find out how many times greater is the population of China than Julie's hometown .

By Question :-

[tex]\rm\implies Population_{China}= 1.4\times 10^9 [/tex]

[tex]\rm\implies Population_{Hometown}= 7,000 [/tex]

Times by which population of China is more:-

For finding this divide the population of China by population of hometown .

[tex]\rm\implies \dfrac{ 1.4 \times 10^9 }{7000} \\\\\rm\implies \dfrac{ 14 \times 10^8}{7\times 10^3}\\\\\rm\implies 2\times 10^{8-3} \\\\\rm\implies 2\times 10^5 \\\\\rm\implies \boxed{\quad \bf 200,000 \quad }[/tex]

Therefore the population of China is greater than 200,000 times .

What is the range of f(x)=4^x

Answers

Answer:

B

Step-by-step explanation:

At - infinity, the function will tend to 0 and at +infinity, the function will tend to +infinity. Those are the two extremas of the function and extremas define the range. Range is all positive real number

Find the measure of the missing angle using the exterior angle sum theorm.

Answers

Answer:

95 degrees

Step-by-step explanation:

First find the angles within the triangle. 180 - 43 - 52 = 85, so the missing angle within the triangle is 85. The exterior angle would be supplementary to that, so 180 - 85 = 95.

Aisha wants to paint the four walls of her living room.
Each wall is 2.2 m high and 5.5 m long.
One wall has a door of 1.8 m by 0.9 m.
Tins of paint cost £13 per 2 L tin.
Each litre of paint can cover 8 m2 of wall.
There is an offer of: Buy 2 tins get the 3rd at half price.
How much will Aisha pay to paint her living room?

Answers

Answer:

£32.50

Step-by-step explanation:

my first question to the teacher : so, no windows in the living room ?

so, it is a square living room with 5.5 m side length.

but each wall is a rectangle of 2.2 × 5.5 m.

for one wall we have to deduct a door area of 1.8×0.9 m.

so, one wall

2.2 × 5.5 = 12.1 m²

4 walls

4 × 12.1 = 48.4 m²

minus one door area

1.8 × 0.9 = 1.62 m²

48.4 - 1.62 = 46.78 m² total paint area

1 L paint covers 8 m².

so, we need 46.78/8 = 5.85 liters.

she gets the paint in 2 L tins. so, she needs 3 tins (6 L).

each tin costs £13.

and because she buys 3 tins, she gets the third one for half the price (13/2 = £6.50).

so, she has to pay

2×13 + 6.50 = 26 + 6.50 = £32.50

What is 8% of $600?
pls provide equation on how you get the answer

Answers

Answer:

Step-by-step explanation:

8% = 8/100

So 8% of 600

= (8/100)* 600

= 8 * 6

= $48.

Step-by-step explanation:

8/100×$600

8×6. (Cancelled 600 by 100%)

$48

hope it helps

PLEASE HELPPPPP, what is the meaning of 1.5 terms context?

Answers

Answer: #2 that's an absolute anser i am not sure if there are more or if this is just a multiple choice question

Step-by-step explanation:

I am having trouble with this problem how do i solve it.

Answers

Answer:

90

Step-by-step explanation:

I do not know if this is true or not but i am pretty sure this is perimeter

All triangles sides have to add up to 180

1. add up 25 and 65

2. 180-90=90

Then i think your answer is 90

Please do not give me hate if i did this wrong


use a double angle or half angle identity to find the exact value of each expression

Answers

Answer:

Step-by-step explanation:

There are 2 very distinct and important things that we need to know before completing the problem. First is that we are given that the cos of an angle is 1/3 (adjacent/hypotenuse) and it is in the first quadrant. We also need to know that the identity for sin2θ = 2sinθcosθ.

We already know cos θ = 1/3, so we need now find the sin θ. The sin ratio is the side opposite the angle over the hypotenuse, and the side we are missing is the side opposite the angle (we do not need to know the angle; it's irrelevant). Set up a right triangle in the first quadrant and label the base with a 1 (because the base is the side adjacent to the angle), and the hypotenuse with a 3. Find the third side using Pythagorean's Theorem:

[tex]3^2=1^2+y^2[/tex] which simplifies to

[tex]9=1+y^2[/tex] and

[tex]y^2=8[/tex] so

[tex]y=\sqrt{8}=2\sqrt{2}[/tex] so that's the missing side. Now we can easily determine that

[tex]sin\theta=\frac{2\sqrt{2} }{3}[/tex]

Now we have everything we need to fill in the identity for sin2θ:

[tex]2sin\theta cos\theta=2(\frac{2\sqrt{2} }{3})(\frac{1}{3})[/tex] and multiply all of that together to get

[tex]2sin\theta cos\theta=\frac{4\sqrt{2} }{9}[/tex]

please help me its urgent​

Answers

Answer:

number of students who like only iphone is 36...

Step-by-step explanation:

100 is the total number..

from that 40 like both so we subtract..

100-40=60..

60 = 2x + 3x..

60 / 5 = 12..

so numbet who like iphone is 3x which is 3 * 12=36..

Grandma is making a quilt. She has 540 cm of fabric to border the quilt. What is the greatest possible area for the quilt?
Question 1 options:


11 664 cm^2


18225 cm^2


72900 cm^2


291600 cm^2

Show your work:

Answers

Answer:

18225 cm²

Step-by-step explanation:

Divide 540 by 4 to get the length of all sides

540/4 = 135

Square 135 to get the max possible size

135² = 18225

18225 cm²  is the greatest possible area for the quilt.

What is area?

The measurement that expresses the size of a region on a plane or curved surface is called area. Surface area refers to the area of an open surface or the boundary of a three-dimensional object, whereas the area of a plane region or plane area refers to the area of a form or planar lamina.

Given

Divide 540 by 4 to obtain the length of all sides

540/4 = 135

Square 135 to acquire the max possible size

135² = 18225

18225 cm²  is the greatest possible area for the quilt.        

To learn more about area refer to:

https://brainly.com/question/25292087

#SPJ2

Three red balls, 5 green balls and a number of blue balls are put together in a sac. One ball is picked at random from the sac. If the probability of picking a red ball is 1|6, find the a) The number of blue balls in sac. B) the probability of picking a green ball​

Answers

Answer:

total balls = 18 .... 3/x = 1/6

blue = 10 ... 18-(5+3) = 10

p of green = 5/18 = .277

Step-by-step explanation:

What is the solution to this system of linear equations?
2%. + 3y = 3
7x-3y = 24
O (
27)
(3,-21)
O (3-1)
19.0

Answers

Answer:

(3,-1)

Step-by-step explanation:

2x + 3y = 3  -----------(I)

7x - 3y = 24 ----------(II)

Add equation (I) & (II) and y will be eliminated and we can find the value of 'x'

(I)         2x + 3y = 3

(II)       7x   - 3y = 24   {Add}

         9x           = 27   {Divide both sides by 9}

                    x = 27/9

x = 3

Plugin x = 3 in equation (I)

2*3 + 3y = 3

  6 + 3y = 3

Subtract 6 from both sides

 3y = 3 - 6

 3y = -3

Divide both sides by 3

    y = -3/3

y = -1

   

Convert to decimal degrees.

-(167° 31”)

[?]°

Enter your answer with three decimal places.

Answers

Answer:

The angle in decimal form is 167.009°.

Step-by-step explanation:

We know an angle in terms of integer angles, minutes and seconds, whose conversion into decimal degrees is expressed by the following formula:

[tex]\theta = n + \frac{m}{60}+\frac{s}{3600}[/tex] (1)

Donde:

[tex]n[/tex] - Integer angle, in sexagesimal degrees.

[tex]m[/tex] - Minutes.

[tex]s[/tex] - Seconds.

If we know that [tex]n = 167[/tex], [tex]m = 0[/tex] and [tex]s = 31''[/tex], then the angle in decimal form is:

[tex]\theta = 167^{\circ}+\frac{0}{60}^{\circ} + \frac{31}{3600}^{\circ}[/tex]

[tex]\theta = 167.009^{\circ}[/tex]

The angle in decimal form is 167.009°.

Find the length of the segment indicated. Round your answer to the nearest 10th if necessary.

Answers

Answer:

x=13.6

Step-by-step explanation:

By Pythagoras theorem, 5.5^2+x^2=14.7^2. x^2=14.7^2-5.5^2. x=13.6

Find cosθ+cos3θ+cos5θ+cos7θ by using the Sum-to-Product Formula.
Please also show your work as well. Thanks!

Answers

Answer:

[tex] \rm\displaystyle 4\cos( \theta) \cos \left( {2\theta} \right) \cos \left( {4 \theta } \right) [/tex]

Step-by-step explanation:

I assume the question want us to rewrite cosθ+cos3θ+cos5θ+cos7θ by using Sum-to-Product Formula and note that it's not an equation therefore θ can never be specified

===========================

so we want to rewrite cosθ+cos3θ+cos5θ+cos7θ by using Sum-to-Product Formula the good news is that the number of the function of the given expression is even so there's a way to do so, rewrite the expression in parentheses notation:

[tex] \rm\displaystyle \left( \cos( \theta) + \cos(3 \theta) \right) + \left(\cos(5 \theta) + \cos(7 \theta) \right)[/tex]

recall that,Sum-to-Product Formula of cos function:

[tex] \rm \boxed{\displaystyle \cos( \alpha ) + \cos( \beta ) = 2 \cos \left( \frac{ \alpha + \beta }{2} \right) \cos \left( \frac{ \alpha - \beta }{2} \right) }[/tex]

notice that we have two pair of function with which we can apply the formula thus do so,

[tex] \rm\displaystyle \left( 2\cos \left( \frac{ \theta + 3 \theta}{2} \right)\cos \left( \frac{ \theta - 3 \theta}{2} \right) \right) + \left(2\cos \left( \frac{5 \theta + 7 \theta}{2} \right) \cos \left( \frac{5 \theta - 7 \theta}{2} \right) \right)[/tex]

simplify addition:

[tex] \rm\displaystyle \left( 2\cos \left( \frac{4 \theta}{2} \right)\cos \left( \frac{ - 2\theta }{2} \right) \right) + \left(2\cos \left( \frac{12 \theta }{2} \right) \cos \left( \frac{ - 2 \theta}{2} \right) \right)[/tex]

simplify division:

[tex] \rm\displaystyle \left( 2\cos \left( {2 \theta} \right)\cos \left( { - \theta } \right) \right) + \left(2\cos \left( {6 \theta } \right) \cos \left( { - \theta} \right) \right)[/tex]

By Opposite Angle Identities we acquire:

[tex] \rm\displaystyle \left( 2\cos \left( {2 \theta} \right)\cos \left( { \theta } \right) \right) + \left(2\cos \left( {6 \theta } \right) \cos \left( { \theta} \right) \right)[/tex]

factor out 2cosθ:

[tex] \rm\displaystyle 2 \cos( \theta) (\cos \left( {2 \theta} \right) + \cos \left( {6 \theta } \right) )[/tex]

once again apply Sum-to-Product Formula which yields:

[tex] \rm\displaystyle 2 \cos( \theta) (2\cos \left( {4\theta} \right) \cos \left( {2 \theta } \right) )[/tex]

distribute:

[tex] \rm\displaystyle 4\cos( \theta) \cos \left( {2\theta} \right) \cos \left( {4 \theta } \right) [/tex]

and we're done!

please help me with this problem !

Answers

Answer: 2/3

Step-by-step explanation:

1/4 (3/12) plus 5/12 = 8/12

which can be simplified by 4

Find cos 0
A. 15/8
B. 15/17
C. 8/15
D. 8/17

Answers

Answer:

A.15/8

Step-by-step explanation:

the answer is 15/8

Answer:

D.

[tex]{ \tt{ \cos( \theta) = \frac{adjacent}{hypotenuse} }} \\ \\ { \tt{ \cos( \theta) = \frac{8}{ \sqrt{ {15}^{2} + {8}^{2} } } }} \\ \\ { \tt{ \cos( \theta) = \frac{8}{ \sqrt{289} } }} \\ \\ { \tt{ \cos( \theta) = \frac{8}{17} }}[/tex]

What is the circumference of a circle with a diameter of 5 feet? Use 3.14 for pie.

Answers

Answer:

A. 15.7 ft

Step-by-step explanation:

circumference = 2πr

radius= half the diameter 5/2=2.5

2(3.14)(2.5)=15.7 ft

If f is continuous for all x, which of the following integrals necessarily have the same value?

Answers

Answer:

B

Step-by-step explanation:

Given the three integrals, we want to determine which integrals necessarily have the same value.

We can let the first integral be itself.

For the second integral, we can perform a u-substitution. Let u = x + a. Then:

[tex]\displaystyle du = dx[/tex]

Changing our limits of integration:

[tex]u_1=(0)+a=a \text{ and } u_2 = (b+a)+a = b+2a[/tex]

Thus, the second integral becomes:

[tex]\displaystyle \int_{0}^{b+a}f(x+a)\, dx = \int_a^{b+2a} f(u)\, du[/tex]

For the third integral, we can also perform a u-substitution. Let u = x + c. Then:

[tex]\displaystyle du = dx[/tex]

And changing our limits of integration:

[tex]\displaystyle u_1=(a-c)+c=a \text{ and } u_2=(b-c)+c=b[/tex]

Thus, our third integral becomes:

[tex]\displaystyle \int_{a-c}^{b-c}f(x+c)\, dx = \int_{a}^{b} f(u)\, du[/tex]

Since the only difference between f(x) and f(u) is the variable and both the first and third integral have the same limits of integration, our answer is B.

Please hurry I will mark you brainliest

Answers

Answer:

To find width when given perimeter and length you must multiply the length by two and subtract that amount from the perimeter. Then divide the new number by two.

Step-by-step explanation:

In a quadrilateral HELP, HE is congruent to LP. What other condition would prove that HELP is a parallelogram?

Answers

Answer:

[tex]HE || LP[/tex]

Step-by-step explanation:

Given

[tex]HE \cong LP[/tex]

Required

Which proves that [tex]HELP[/tex] is a parallelogram

We have:

[tex]HE \cong LP[/tex]

The given sides must also be parallel i.e.

[tex]HE || LP[/tex]

Hence, (c) is correct

Other Questions
What explains Louis Armstrong's significance to jazz during the Harlem Renaissance? He was the first recorded jazz artist and considered the best jazz composer. He was a leading saxophonist and toured all over the world with his big band. He released more albums than any other jazz artist, both solo and with his big band. He was the first great jazz soloist and made movie, radio, and television appearances. Find the x- and y-intercepts of the following line: 4x 3y = 12 Need help urgent answering this question Which diagram best shows how fraction bars can be used to evaluate One-half divided by one-fourth? A fraction bar labeled 1. Under the 1 are 2 boxes containing one-half. Under the 2 boxes are 4 boxes containing one-fourth. 2 one-fourths are circled. A fraction bar labeled one-fourth. Under the one-fourth are 2 boxes containing one-half. Under the 2 boxes are 4 boxes containing 1. 2 boxes containing 1 are circled. A fraction bar labeled 1. Under the 1 are 4 boxes containing one-fourth. Under the 4 boxes are 8 boxes containing one-half. One box containing one-half is circled. A fraction bar labeled 1. Under the 1 are 4 boxes containing one-fourth. Under the 4 boxes are 2 boxes containing one-half. One box containing one-half is circled. Aqueous sulfuric acid will react with solid sodium hydroxide to produce aqueous sodium sulfate and liquid water . Suppose 2.9 g of sulfuric acid is mixed with 3.53 g of sodium hydroxide. Calculate the maximum mass of water that could be produced by the chemical reaction. Round your answer to significant digits. The table below shows some inputs and outputs of the invertible function f with domain all real numbers. PLEASE HELP ME!!!!! ( photo given ) What do LinkedIn automation tools do? The Ottoman, Safavid, and Mughal were formed as Islamic empires after the spread of Islam. Please select the best answer from the choices provided T F Scenario: Over the last several months, there has been a rapid increase in the number of loans that banks have provided for mortgages and small businesses. This change has raised concerns for the Fed. Today, the Fed has announced an increase in the interest rates that it is charging banks. In this scenario, what is the Fed trying to do by increasing interest rates 5 square root 36^4=36 true or false What is conflict? Explain how it affects the action of a storyPLS HELP IN QUIZ!!WILL MARK BRAINLIEST!!25 POINTS!! Why did Marxists come to believe that this distinction was paramount for understanding history and shaping the future? Chromium-51 is a radioisotope that is used to assess the lifetime of red blood cells The half-life of chromium-51 is 27.7 days. If you begin with 39.7 mg of this isotope, what mass remains after 48.2 days have passed? SOMEONE HELP ME PLEASEDecide if the following scenario involves a permutation or combination. Then find the number of possibilities.You are setting the combination on a five-digit lock. You want to use the numbers 12345 but you don't care what order they are in. In order to dry wet clothes, we spread them on a clothline. This is because (i) spreading increases surface area (ii) clothes become brighter when spread change into indirect speech (1) She said,"what does BOB do?" How many moles of gas occupy a volume of 101.3L? Work out 45% of $200.00 Change the following into the active voice: Forsyth Company manufactures one product, it does not maintain any beginning or ending inventories, and its uses a standard cost system. During the year, the company produced and sold 10,000 units at a price of $155 per unit. Its standard cost per unit produced is $125 and its selling and administrative expenses totaled $245,000. Forsyth does not have any variable manufacturing overhead costs and it recorded the following variances during the year:Materials price variance $6,800 FMaterials quantity variance $10,500 ULabor rate variance $3,800 ULabor efficiency variance $4,700 FFixed overhead budget variance $2,800 FFixed overhead volume variance $12,300 FRequired:a. When Forsyth closes its standard cost variances, the cost of goods sold will increase (decrease) by how much?b. Prepare an income statement for the year.